A really interesting floor problem!

  Рет қаралды 16,941

Michael Penn

Michael Penn

2 жыл бұрын

Suggest a problem: forms.gle/ea7Pw7HcKePGB4my5
Please Subscribe: kzbin.info...
Merch: teespring.com/stores/michael-...
Personal Website: www.michael-penn.net
Randolph College Math: www.randolphcollege.edu/mathem...
Randolph College Math and Science on Facebook: / randolph.science
Research Gate profile: www.researchgate.net/profile/...
Google Scholar profile: scholar.google.com/citations?...
If you are going to use an ad-blocker, considering using brave and tipping me BAT!
brave.com/sdp793
Buy textbooks here and help me out: amzn.to/31Bj9ye
Buy an amazon gift card and help me out: amzn.to/2PComAf
Books I like:
Sacred Mathematics: Japanese Temple Geometry: amzn.to/2ZIadH9
Electricity and Magnetism for Mathematicians: amzn.to/2H8ePzL
Abstract Algebra:
Judson(online): abstract.ups.edu/
Judson(print): amzn.to/2Xg92wD
Dummit and Foote: amzn.to/2zYOrok
Gallian: amzn.to/2zg4YEo
Artin: amzn.to/2LQ8l7C
Differential Forms:
Bachman: amzn.to/2z9wljH
Number Theory:
Crisman(online): math.gordon.edu/ntic/
Strayer: amzn.to/3bXwLah
Andrews: amzn.to/2zWlOZ0
Analysis:
Abbot: amzn.to/3cwYtuF
How to think about Analysis: amzn.to/2AIhwVm
Calculus:
OpenStax(online): openstax.org/subjects/math
OpenStax Vol 1: amzn.to/2zlreN8
OpenStax Vol 2: amzn.to/2TtwoxH
OpenStax Vol 3: amzn.to/3bPJ3Bn
My Filming Equipment:
Camera: amzn.to/3kx2JzE
Lense: amzn.to/2PFxPXA
Audio Recorder: amzn.to/2XLzkaZ
Microphones: amzn.to/3fJED0T
Lights: amzn.to/2XHxRT0
White Chalk: amzn.to/3ipu3Oh
Color Chalk: amzn.to/2XL6eIJ

Пікірлер: 64
@demenion3521
@demenion3521 2 жыл бұрын
You could also just end the first inclusion at sqrt(4n+2). This would in the end yield the inequality 0
@Czeckie
@Czeckie 2 жыл бұрын
yep, the discussion about 4n+2, 4n+3 added nothing
@matteoanoffo1447
@matteoanoffo1447 2 жыл бұрын
It's the second time he does this exercise in his videos, nice
@somasahu1234
@somasahu1234 2 жыл бұрын
Understood everything except the 1st equivalent statement !
@fix5072
@fix5072 2 жыл бұрын
It is about the rhs of the goal equation. The expression inside the sqrt must be strictly less then the next bigger perfect square (4n+4) and strictly bigger than the next lower perfect square (4n+1). Thus the lhs must also be on that intervall
@ConManAU
@ConManAU 2 жыл бұрын
@@fix5072 Not that 4n+1 and 4n+4 are guaranteed to be perfect squares, but 4n+2 and 4n+3 absolutely cannot, so if floor(sqrt(4n+2))=k, then k^2
@fejfo6559
@fejfo6559 2 жыл бұрын
the squares mod 4 are 0 and 1, so 4n+2 isn't a perfect square. That means 4n+2 is in between 2 perfect squares a² < 4n+2 < (a+1)². If we take the square root the inequality we see floor(4n+2) = a. So the goal becomes sqrt(n)+sqrt(n+1) in [a, a+1) Because of the squares mod 4, a² < 4n+1 and 4n+4 < (a+1)². So his statement suffices.
@fix5072
@fix5072 2 жыл бұрын
@@fejfo6559 this is what i was trying to say, even tho the inequalites must be a^2
@tonyhaddad1394
@tonyhaddad1394 2 жыл бұрын
Yes michael always skip many steps
@kevinmartin7760
@kevinmartin7760 2 жыл бұрын
I'm having a little trouble with the first step, especially the "equivalency" claim. What is not clear to me is that the goal implies the first step, which is one of the requirements for equivalency. Maybe is does (but it is not obvious to show), or maybe it doesn't, but then that doesn't matter as equivalency is not required; all that is required is that the first step imply the goal.
@henryhowe769
@henryhowe769 2 жыл бұрын
Perhaps it would help to understand this in terms of inequalities instead first a few examples of the floor function floor(3.2) = 3 because 3.2 ∈ (3,4) which is equivalent to saying that 3 < 3.2 < 4 and 3 is the lowest value in this interval floor(5.9) = 5 because 5.9 ∈ (5,6) which is equivalent to saying that 5 < 5.9 < 6 and 5 is the lowest value in this interval we are looking at the floor(√[4n +2]) so what is the integer interval that we should be working with? well for a square root to be an integer the input must be a perfect square, and the nearest values that meet this requirement relative to √[4n+2] are √[4n+1] & √[4n+4] In other words √[4n+1] < √[4n+2] < √[4n+4] our claim is that floor(√[n] + √[n+1]) = floor(√√[4n+2]) this implies that they are on the same interval therefore we can substitute and say that √[4n+1] < √[n] + √[n+1] < √[4n+4] or in other words √[n] + √[n+1] ∈ (√[4n+1],√[4n+4]) Hope that helps :)
@E1Luch
@E1Luch 2 жыл бұрын
@@henryhowe769 but why √[4n+1] possibly being an integer would imply that it is smaller than √[n] + √[n+1] for all n? it is smaller in the end, but its not always an integer, and if understand this correctly it was only proven at the end of the video, directly, without the requirement of the original goal equality, and then the goal was implied from it, and only then we could say that these statements are equivalent because they are both true for any natural n.
@coc235
@coc235 2 жыл бұрын
@@E1Luch Right. The inclusion implies the goal equation, but not vice versa
@henryhowe769
@henryhowe769 2 жыл бұрын
​@@E1Luch √[4n+1] possibly being an integer does not imply, that it is smaller than √[n] + √[n+1] if it did the rest of the proof would be unnecessary. Rather the goal equation suggests that such inequality might be true. Its more like an educated guess.
@balthazarbeutelwolf9097
@balthazarbeutelwolf9097 2 жыл бұрын
first step is just wrong. From the original you get strict a lower bound of sqrt(4n+2)-1 and a strict upper bound of sqrt(4n+2)+1. One proves these inequations by a sequence of logical steps mostly involving squaring both sides and getting rid of common terms. Looks nothing like Michael's proof though.
@manucitomx
@manucitomx 2 жыл бұрын
Thank you, professor!
@Kosemagician
@Kosemagician 2 жыл бұрын
2:31 if n=0 it is in fact possible for sqrt(n)+sqrt(n+1) to be a natural number, so the lower bound should be included [sqrt(4n+1), sqrt(4n+4)) All of this depends on whether you define 0 to be a natural number in the first place
@luisaleman9512
@luisaleman9512 2 жыл бұрын
Michael doesn't consider 0 to be a natural number.
@sugongshow
@sugongshow 2 жыл бұрын
Just like Michael doesn't consider 0 to be a natural number, the rest of the world should not either as natural numbers are 1, 2, 3, 4, 5, .......
@balthazarbeutelwolf9097
@balthazarbeutelwolf9097 2 жыл бұрын
@@sugongshow In Computer Science everyone considers 0 to be a natural number. When you do number theory modulo n you simply need 0. Of course, it's just a definition, just like excluding 1 from being prime - it's just a case which "works better" in the part of maths you are interested in. But Peano arithmetic uses 0 as the base case too.
@stewartcopeland4950
@stewartcopeland4950 2 жыл бұрын
Good Place To Stop preferred to take a nap
@goodplacetostop2973
@goodplacetostop2973 2 жыл бұрын
I was outside for a visa renewal actually. The nap will be later 😂
@klausg1843
@klausg1843 2 жыл бұрын
I miss an explanation of the conclusive step. So I propose the following: Since both n^0.5+(n+1)^0.5 and (4n+2)^0.5 lie in the open interval from (4n+1)^0.5 to (4n+4)^0.5 and this interval does not contain an integer, then the conclusion follows.
@sugongshow
@sugongshow 2 жыл бұрын
I love that shirt!!
@wesleydeng71
@wesleydeng71 2 жыл бұрын
First, LHS < RHS. So we only need to consider the case sqrt(n+1)=k/2 (where LHS is most close to an integer k but still less than it). In this case, 4n+2=k^2-2, which means sqrt(4n+2) is also less than k.
@ByteOfCake
@ByteOfCake 2 жыл бұрын
Have you ever thought of doing videos on the derivation of the explicit formula of the prime counting function? Like finding identities for ln(zeta(s)) and then using Mellin inversion. I think it would be interesting.
@ByteOfCake
@ByteOfCake 2 жыл бұрын
actually, i'll put it in your form. Didn't see it
@minamagdy4126
@minamagdy4126 2 жыл бұрын
I wonder if your proof also shows that both also equals floor(sqrt(4n+3)), which makes sense given the between 2 integers thing in the top line. Also would be interesting to explore all situations for which 4n+1 or 4n+4 are not equal to the above (after the transformation. I expect low numbers like 1 to be the only exceptions.
@jabunapg1387
@jabunapg1387 2 жыл бұрын
This Problem was in the first round of the german Math Olympiad a few years ago
@dicksonchang6647
@dicksonchang6647 2 жыл бұрын
same problem?
@pranjalpathak214
@pranjalpathak214 2 жыл бұрын
Wow nice job for making the complex thing more complex🤣🤣🤣
@cyberjet5946
@cyberjet5946 2 жыл бұрын
Can we do it with AM >= GM property? sqrt(n) + sqrt(n+1) >= 2 * sqrt(sqrt(n).sqrt(n+1)) I have arrived till sqrt(n) + sqrt(n+1) >= sqrt(4n + 2). How to proceed next? is this possible via this method? How do I apply the floor function on this inequality logically if possible?
@drsonaligupta75
@drsonaligupta75 2 жыл бұрын
8:21
@user-ur2en1zq4f
@user-ur2en1zq4f 2 жыл бұрын
Where is that good place to buy that good shirt?
@Frank9412co
@Frank9412co 2 жыл бұрын
In the description there is the 🔗 to the teespring site
@user-ur2en1zq4f
@user-ur2en1zq4f 2 жыл бұрын
@@Frank9412co where is that good place you live?
@ayoubabid8783
@ayoubabid8783 2 жыл бұрын
Good
@user-bx7rw1pt4p
@user-bx7rw1pt4p 2 жыл бұрын
I didn't understand why the inclusion on the interval solve the problem. For example: if the sum is a little bit greater than sqrt(4n+1) then the floor of the sum is equal to the floor of sqrt(4n+1)
@deadfish3789
@deadfish3789 2 жыл бұрын
But the floor of sqrt(4n+1) is always equal to the floor of sqrt(4n+2). If not, that would be equivalent to saying there is a square number on (4n+1, 4n+2], but clearly this is impossible since they are consecutive integers and there are no squares of the form 4n+2.
@user-bx7rw1pt4p
@user-bx7rw1pt4p 2 жыл бұрын
@@deadfish3789 thanks
@chessematics
@chessematics 2 жыл бұрын
Meanwhile Ceiling: becomes jealous of his brother.
@ZedaZ80
@ZedaZ80 2 жыл бұрын
You should remake all of those videos, but replace floor(x) with -ceiling(-x) XD
@backyard282
@backyard282 2 жыл бұрын
But why you never show any love for the ceiling function :(
@theophonchana5025
@theophonchana5025 2 жыл бұрын
(Square root of n) + square root of (n+1) = Square root of (4n+2) n = undefined
@romajimamulo
@romajimamulo 2 жыл бұрын
There's something I find kinda weird about the range you picked. It's not clear to me that it will always be tighter than the range needed for the floor function
@leif_p
@leif_p 2 жыл бұрын
He went very quickly through that part, but his range is equivalent to asking "What are the closest numbers to 4n + 2 that _could_ be the square of an integer?" This is actually _stronger_ than needed for the floor function for large n. Plug in n=100 and you get (approximately): 20.02 < sqrt(4n+2) < 20.07, way tighter than needed.
@romajimamulo
@romajimamulo 2 жыл бұрын
@@leif_p right, I knew it was smaller, I just wasn't sure about the fact it has to be between the integers.
@anguzman
@anguzman 8 ай бұрын
I saw this problem as #20 of Chapter 3 of Tom Apostol's number theory book.
@MrDivinity22
@MrDivinity22 2 жыл бұрын
From the proof it looks like this equality also holds with 4n+3 on the right, doesn't it?
@binaryagenda
@binaryagenda 2 жыл бұрын
And with 4n + 1. It's kind of unsatisfying that it is not a unique expression.
@user-bx7rw1pt4p
@user-bx7rw1pt4p 2 жыл бұрын
Sqrt(4n+3) can t be a perfect sqr then its floor is equivalent to the floor of 4n+2
@titan1235813
@titan1235813 2 жыл бұрын
No. Okay, yes ☺
@Frank9412co
@Frank9412co 2 жыл бұрын
8:22 is a good place to stop
@goodplacetostop2973
@goodplacetostop2973 2 жыл бұрын
Yes, it is.
@Gandarf_
@Gandarf_ 2 жыл бұрын
That problem already was on this channel more than year ago btw
@TI5040
@TI5040 2 жыл бұрын
Isn't it ramanujan's result?
@idontknowwhatido3972
@idontknowwhatido3972 2 жыл бұрын
Hey, he is behaving quite differently...hm maybe he is drunk?, high? withdrawal?
@trueriver1950
@trueriver1950 2 жыл бұрын
Counter example N=0 LHS = 1 RHS = 2 The failed step in the proof is where Michael assumes that consecutive integers cannot both be squares. Did I miss a comment that n has to be positive excluding zero?
@theophonchana5025
@theophonchana5025 2 жыл бұрын
#undefined
@wojteksocha2002
@wojteksocha2002 2 жыл бұрын
Cool video bro, but I think I have to copystirke your video cause Michael Penn already made video about it.
@trueriver1950
@trueriver1950 2 жыл бұрын
I am guessing he used it with permission :) He did say at the outset that this is a problem he really likes...
@aashutoshgupta6690
@aashutoshgupta6690 2 жыл бұрын
First view
this equation will floor you | 2004 Uzbekistan Math Olympiad
19:56
Michael Penn
Рет қаралды 16 М.
an infinitely long solution.
10:53
Michael Penn
Рет қаралды 23 М.
Inside Out 2: Who is the strongest? Joy vs Envy vs Anger #shorts #animation
00:22
Why Is He Unhappy…?
00:26
Alan Chikin Chow
Рет қаралды 26 МЛН
НЫСАНА КОНЦЕРТ 2024
2:26:34
Нысана театры
Рет қаралды 1 МЛН
A few of my favorite things!
13:06
Michael Penn
Рет қаралды 30 М.
Solving a crazy iterated floor equation.
22:38
Michael Penn
Рет қаралды 141 М.
The Impossible Problem NO ONE Can Solve (The Halting Problem)
20:24
A practice problem from India
10:14
Michael Penn
Рет қаралды 21 М.
The Longest Standing Mathematical Conjecture That Was Actually False
14:08
Why the EU is About to Impose a Hard Border in Gibraltar
9:42
TLDR News
Рет қаралды 476 М.
The Reason Train Design Changed After 1948
13:05
Joe Scott
Рет қаралды 194 М.
a crazy integral - floor function - limit problem
10:17
Maths 505
Рет қаралды 6 М.
Why 7 is Weird - Numberphile
12:03
Numberphile
Рет қаралды 1,8 МЛН
Inside Out 2: Who is the strongest? Joy vs Envy vs Anger #shorts #animation
00:22